LSAT and Law School Admissions Forum

Get expert LSAT preparation and law school admissions advice from PowerScore Test Preparation.

 Administrator
PowerScore Staff
  • PowerScore Staff
  • Posts: 8917
  • Joined: Feb 02, 2011
|
#23685
Complete Question Explanation

Must be True-SN. The correct answer choice is (C)

Terry states that if you want a decent job, you should go to college. Basically:
  • Job ..... :arrow: ..... somewhat necessary ..... :arrow: ..... college
Mark replies that there are other reasons to go to college other than to get a job.

That is an attack on the conditional:
  • College ..... :arrow: ..... Job
Mark has misinterpreted Terry's statement. Terry meant to imply that going to college is recommended, or somewhat necessary and somewhat helpful, if you want a decent job. Mark interprets the statement as the Mistaken Reversal, which is that wanting a decent job is necessary before one would bother attending college.

Since you are asked to identify Mark's interpretation, you should focus on his error.

Answer choice (A): The idea that one can train for a job at many places other than college would not help explain why Mark stated that one can go to college for many reasons other than getting a job, so this choice is wrong.

Answer choice (B): This statement is basically a more definite form of Terry's conditional: "Job >→College." Mark's error is a complete reversal, not one of extremes, so this choice is wrong.

Answer choice (C): This is the correct answer choice. In responding that Terry is wrong because there are in fact many reasons other than a job to go to college, Mark implies that he believes Terry proposed that getting a decent job is the only reason to go to college.

Answer choice (D): Mark's response regards whether there are other things than jobs relevant to college, but this incorrect response regards whether things other than colleges are relevant to jobs.

Answer choice (E): Mark's response concerns whether people who go to college are all there for jobs, but this incorrect response concerns whether all people who want decent jobs go to college.

As a note, this stimulus does contain potential cause-effect propositions, but the fact is that it is much better to discuss the stimulus in terms of necessary and sufficient conditions, because Mark's reply is based on a Mistaken Reversal.

Get the most out of your LSAT Prep Plus subscription.

Analyze and track your performance with our Testing and Analytics Package.